Moyses - Curso de Física Básica, Electromagnetismo , Problema Resolvido 8.5


8.5) Nas figs. (a) e (b), as porções estafiliníneas dos fios são supostas muito longas e a porção semicircular tem raio $R$. A corrente tem intensidade $i$. Calcule o campo $\vec{B}$, em módulo, direção e sentido, no centro $P$ das porções semicirculares, em ambos os casos.

a) A equação de Biot-Savart é dada por $$\vec{B}=\frac{\mu_0i}{4\pi}\int\frac{d\vec{l}\times\vec{r}}{r^3}\ \ \ \ \ (1)$$ Adotando o referencial sobre o ponto onde queremos calcular o campo, o ponto $P$.
Note que a posição $\vec{r}_1$ e $\vec{r}_3$ são paralelos ao elemento de comprimento $d\vec{l}_1$ e $d\vec{l}_3$, das seções retilíneas do fio, são paralelos. Desse modo as partes retilíneas são contribuem com o campo. Desse modo calcularemos apenas o campo devido à parte semi circular. Qualquer ponto do fio semi-circular pode ser localizado com o vetor $\vec{R}=R\cos\theta\hat{i}+R\sin\theta\hat{j}$. Como o referencial esta na origem o vetor $\vec{r}$, com origem no elemento de corrente e extremidade no ponto $P$ onde queremos calcular o campo (origem), é dado por $\vec{r}=-\vec{R}$. O elemento de comprimento $d\vec{l}_2$ é dado pela derivada de $\vec{R}$, isto é, $d\vec{l}_2=\frac{dl_3}{R}\frac{d\vec{R}}{dt}=-dl\sin\theta\hat{i}+dl\cos\theta\hat{j}$. Substituindo essas informações na equação (1) obtemos $$\vec{B}=\frac{\mu_0i}{4\pi}\int\frac{(-dl\sin\theta\hat{i}+dl\cos\theta\hat{j})\times(-R\cos\theta\hat{i}-R\sin\theta\hat{j})}{R^3}$$ O resultado do produto vetorial na expressão é $Rdl\hat{k}$, logo $$\vec{B}=\frac{\mu_0i}{4\pi}\int\frac{Rdl}{R^3}\hat{k}$$ O elemento de comprimento pode ser escrito como $dl=Rd\theta$ $$\vec{B}=\frac{\mu_0i}{4\pi R}\int_{\pi}^{0}d\theta\hat{k}$$ O que resulta em $$\vec{B}=-\frac{\mu_0i}{4R}\hat{k} \ \ \ \ (2)$$ b) As informações a respeito dos vetores que serão usados no item (b) estão sintetizados na figura a baixo.
Os vetores $\vec{r}_1$ e $\vec{r}_3$, são os vetores simétricos dos vetores posições de cada parte retilínea do fio são dados por $\vec{R}_1=x\hat{i}-R\hat{j}$ e $\vec{R}_2=x\hat{i}+R\hat{j}$. A parte semi-circular foi calculado na parte (a) do problema,, dessa forma obtemos que o campo é $$\vec{B}=\frac{\mu_0i}{4\pi}\int\frac{d\vec{l}\times\vec{r_1}}{r_1^3}-\frac{\mu_0i}{4R}\hat{k}+\frac{\mu_0i}{4\pi}\int\frac{d\vec{l}\times\vec{r_3}}{r_3^3}\Rightarrow$$ Os elementos de comprimento são $d\vec{l}=dx\hat{j}$ $$\vec{B}=\frac{\mu_0i}{4\pi}\int_{-\infty}^0\frac{dx\hat{j}\times(-x\hat{i}+R\hat{j})}{[x^2+R^2]^{\frac{3}{2}}}-\frac{\mu_0i}{4R}\hat{k}+\frac{\mu_0i}{4\pi}\int_{-\infty}^0\frac{dx\hat{j}\times(-x\hat{i}-R\hat{j})}{[x^2+R^2]^{\frac{3}{2}}}\Rightarrow$$ $$\vec{B}=\frac{\mu_0i}{4\pi}\int_{-\infty}^0\frac{-x\hat{j}\times\hat{i}+R\hat{j}\times\hat{j}}{[x^2+R^2]^{\frac{3}{2}}}dx-\frac{\mu_0i}{4R}\hat{k}+\frac{\mu_0i}{4\pi}\int_{-\infty}^0\frac{-x\hat{j}\times\hat{i}-R\hat{j}\times\hat{j}}{[x^2+R^2]^{\frac{3}{2}}}dx$$ O produto vetorial $\hat{j}\times\hat{j}$ é nulo enquanto $\hat{j}\times\hat{i}=-\hat{k}$, logo $$\vec{B}=\frac{\mu_0i}{4\pi}\int_{-\infty}^0\frac{x}{[x^2+R^2]^{\frac{3}{2}}}dx\hat{k}-\frac{\mu_0i}{4R}\hat{k}+\frac{\mu_0i}{4\pi}\int_{-\infty}^0\frac{x}{[x^2+R^2]^{\frac{3}{2}}}dx\hat{k}$$ Realizando a dissubstituição $x=R\tan\theta$ obtemos $$\vec{B}=\frac{\mu_0i}{4\pi R}\int_{-\frac{\pi}{2}}^0\sin\theta d\theta\hat{k}-\frac{\mu_0i}{4R}\hat{k}+\frac{\mu_0i}{4\pi R}\int_{-\frac{\pi}{2}}^0\sin\theta d\theta\hat{k}\Rightarrow$$ $$\vec{B}=\frac{\mu_0i}{2\pi R}[-\cos\theta ]_{-\frac{\pi}{2}}^0\hat{k}-\frac{\mu_0i}{4R}\hat{k}\Rightarrow$$ $$\vec{B}=-\frac{\mu_0i}{2\pi R}\hat{k}-\frac{\mu_0i}{4R}\hat{k}\Rightarrow$$ $$\vec{B}=-\frac{\mu_0i}{4R}\left( 1+\frac{2}{\pi}\right) \hat{k}$$

Comentários

Postagens mais visitadas